¿Siempre crece la incertidumbre de Heisenberg bajo la evolución del tiempo?

Recientemente ha habido algunas preguntas interesantes sobre QM estándar y especialmente sobre el principio de incertidumbre y disfruté revisando estos conceptos básicos. Y me di cuenta de que tengo una pregunta interesante propia. Supongo que la respuesta debería conocerse, pero no pude resolver el problema por mí mismo, así que espero que no sea del todo trivial.

Entonces, ¿qué sabemos sobre el error de la medición simultánea bajo la evolución del tiempo? Más precisamente, ¿es siempre cierto que para t 0

X ( t ) 2 pags ( t ) 2 X ( 0 ) 2 pags ( 0 ) 2
(aquí argumento ( t ) denota expectativa en estado evolucionado ψ ( t ) , o de manera equivalente para el operador en la imagen de Heisenberg).

Traté de obtener límites generales de la ecuación de Schrödinger y la descomposición en estados propios de energía, etc., pero no veo ninguna forma de probarlo. Sé que esta afirmación es cierta para un paquete de ondas gaussianas gratuito. En este caso, obtenemos igualdad, de hecho (porque el paquete se mantiene gaussiano y porque minimiza HUP). Creo que esto es, de hecho, lo mejor que podemos obtener y para otras distribuciones obtendríamos una desigualdad estricta.

Entonces, para resumir las preguntas.

  1. ¿Es verdadera la afirmación?
  2. Si es así, ¿cómo se prueba? ¿Y hay una manera intuitiva de ver que es verdad?
¿Por qué crees que se aplicaría? Realmente no puede hacer una medición de esa manera (o mide en t = 0 o en t = T , pero nunca ambos), por lo que básicamente tienes dos ψ soluciones Ambos obedecerán el principio de forma independiente. ¿Estoy malinterpretando tu pregunta?
Si su paquete de ondas, para empezar, satura el límite de incertidumbre (es decir, es un estado coherente), entonces esto es trivialmente cierto: los estados coherentes permanecen coherentes bajo la evolución del tiempo. Si su estado inicial no es un estado coherente, entonces la evolución está claramente más involucrada, pero en ese caso podría expandir su estado inicial arbitrario en la base del estado coherente, de modo que esta desigualdad (establecida para estados coherentes) aún podría usarse, componente por componente para mostrar que sigue siendo cierto para el estado arbitrario. O tal vez no. Chug y plug, nena, chug y plug.
@Sklivvz: no hay problema con eso. La partícula aún necesita satisfacer HUP en cada momento, incluso si no lo mide; Solo quiero hacer esta afirmación cuantitativa. Si ayuda, piense en esto como un problema puramente matemático.
@Deepak: buena idea. Sé que los estados coherentes son útiles para el oscilador armónico y supongo que sus perturbaciones también. Pero ¿qué pasa con un sistema general? ¿Hay siempre estados coherentes presentes?
@Marek, entiendo, sin embargo, su declaración es más fuerte que HUP, ¿no es así?
@Sklivvz: hm, no realmente más fuerte. Es independiente pero (si es cierto) da más información sobre el comportamiento de la incertidumbre.
@Marek, los estados coherentes no necesitan estar "presentes" para usarlos en un cálculo centrado en la base. Recuerde que CS forma una base sobrecompleta para el espacio de Hilbert.
No creo que la afirmación sea cierta. Ponga el paquete de ondas de incertidumbre mínima en t=0. ¿Cuál era la incertidumbre antes, en t<0? era más grande por lo que ha estado disminuyendo antes de t=0. De manera más general, no puede derivar declaraciones asimétricas en el tiempo a partir de leyes simétricas en el tiempo.
@Roy: corríjame si me equivoco, pero supuse que los estados coherentes son estados especiales que satisfacen ciertas condiciones (a saber, la minimización de HUP) en todo momento. Es obvio que esta condición depende del hamiltoniano preciso y no es obvio para mí que tales estados siempre se puedan encontrar. ¿Quizás estás hablando de CS de oscilador armónico? Si es así, ¿cómo me ayudan estos? Seguramente no son conservados por la evolución bajo un hamiltoniano arbitrario.
@Moshe: hay lagunas en su argumento: puede que no haya un mínimo para un sistema dado (solo un mínimo) y si hay un mínimo, podría conservarse en la evolución (como para Gaussian libre). Aún así, buena idea e intentaré usarla para encontrar un contraejemplo en algún sistema simple. En cuanto a la segunda declaración: correcto, así que estoy seguro de que me dirá que no podemos obtener la segunda ley también... es broma, no quiero entrar en esta discusión que hizo que Boltzmann se suicidara :)
@Marek, en cualquier ejemplo que pueda resolver la ecuación de Schrodinger, encontrará que la cantidad que le interesa crece alejándose de t = 0, tanto hacia el pasado como hacia el futuro, esto está garantizado por la simetría. En cuanto al enunciado general, también es cierto para la segunda ley. No puede derivar conclusiones asimétricas en el tiempo a partir de leyes simétricas en el tiempo sin una entrada adicional, esto es solo lógica básica, nada que ver con la física. Toda la discusión es qué es esa entrada adicional y dónde entra.
@Marek, aún no he probado el cálculo sugerido por Deepak (demasiados Stack Q para revisar), pero una base es solo eso. Los vectores base x son las funciones delta de posición, p la base delta del momento (con una proporción dada de cada valor para Ψ ), así como la base CS. El problema es que está demasiado completo, por lo que podría causar problemas, es decir Ψ ( t ) = a 1 ( t ) C S 1 + a 2 ( t ) C S 2 + . . . .
@Moshe: buenos puntos, gracias. @Roy: entiendo que es un conjunto de vectores que abarcan el espacio de Hilbert, pero la pregunta es, ¿cómo se definen para el sistema general? ¿Siempre de la misma manera (es decir, como "estados propios" de los operadores de aniquilación) o dependiendo del hamiltoniano? Nunca los he encontrado además de la clase QM estándar cuando hablo del oscilador armónico, por lo que no tengo idea de la situación general.
@Marek: Hay muchas formas de generalizar estados coherentes a sistemas no armónicos. 1) Estados coherentes del operador de aniquilación. 2) Estados coherentes del grupo de simetría dinámica (coset G/H donde H es el grupo de estabilidad del vector fiduciario). 3) Estados de incertidumbre mínimos (e iguales). 4) Estado coherente generalizado de Klauder.... Todas las definiciones coinciden para el oscilador armónico y pueden extenderse para obtener estados comprimidos generalizados. Defn 3) puede no ser estable en el tiempo, según los observables que esté minimizando. Véase también nLab
@Simon: muy interesante, no tenía idea de que había tantas definiciones. ¿Podría mencionar también algunas aplicaciones útiles (si conoce algunas)?
@Marek: Realmente no soy de los que preguntan por las aplicaciones... Klauder (y otros) las usa para su enfoque alternativo de las integrales de ruta y la cuantificación. El último tercio del libro de Perelemov está dedicado a las aplicaciones físicas. Puedes mirar el Contenido para ver qué.
@Marek: miré estas cosas hace mucho tiempo como un proyecto de cuarto año, y la única aplicación que realmente miré fue la aproximación de soluciones clásicas. Ahora, mirando hacia atrás, entiendo mucho más, y veo que hay algunas cosas relacionadas con mi trabajo actual que realmente debería mirar más de cerca...
Personalmente, creo que el libro de Ali, Antoine y Gazeau es una gran referencia para los estados coherentes. Las primeras páginas por sí solas son suficientes para dar a la mayoría de las personas todo lo que necesitan para lidiar con estados coherentes. En cuanto a las aplicaciones, los estados comprimidos (generalizaciones de estados coherentes) se utilizan con frecuencia en cosmología , en particular, para abordar cuestiones sobre el surgimiento de la clasicidad después de la inflación.

Respuestas (6)

La pregunta se refiere a la dependencia del tiempo de la función.

F ( t ) := ψ ( t ) | ( Δ X ^ ) 2 | ψ ( t ) ψ ( t ) | ( Δ pags ^ ) 2 | ψ ( t ) ,

dónde

Δ X ^ := X ^ ψ ( t ) | X ^ | ψ ( t ) , Δ pags ^ := pags ^ ψ ( t ) | pags ^ | ψ ( t ) , ψ ( t ) | ψ ( t ) = 1.

Aquí usaremos la imagen de Schrödinger donde los operadores son constantes en el tiempo, mientras que los kets y los bras evolucionan.

Editar : Estimulado por los comentarios de Moshe R. y Ted Bunn, agreguemos que (bajo el supuesto (1) a continuación) la ecuación de Schroedinger en sí misma es invariante bajo el operador de inversión de tiempo T ^ , que es un operador lineal conjugado, de modo que

T ^ t = t T ^ , T ^ X ^ = X ^ T ^ , T ^ pags ^ = pags ^ T ^ , T ^ 2 = 1.

Aquí nos estamos restringiendo a los hamiltonianos. H ^ de modo que

[ T ^ , H ^ ] = 0. ( 1 )

Además, si

| ψ ( t ) = norte ψ norte ( t ) | norte

es una solución a la ecuación de Schrödinger en cierta base | norte , después

T ^ | ψ ( t ) := norte ψ norte ( t ) | norte

también será una solución a la ecuación de Schrödinger con una función reflejada en el tiempo F ( t ) .

Así si F ( t ) no es constante en el tiempo, entonces podemos suponer (posiblemente después de una operación de inversión del tiempo) que existen dos tiempos t 1 < t 2 con F ( t 1 ) > F ( t 2 ) . Esto contradiría la declaración en la pregunta original. Para finalizar el argumento, proporcionamos a continuación un ejemplo de una función no constante F ( t ) .

Considere un oscilador armónico simple hamiltoniano con energía de punto cero 1 2 ω restado para conveniencia posterior.

H ^ := pags ^ 2 2 metro + 1 2 metro ω 2 X ^ 2 1 2 ω = ω norte ^ ,

dónde norte ^ := a ^ a ^ es el operador numérico.

Pongamos las constantes metro = = ω = 1 a uno por simplicidad. Entonces los operadores de aniquilación y creación son

a ^ = 1 2 ( X ^ + i pags ^ ) , a ^ = 1 2 ( X ^ i pags ^ ) , [ a ^ , a ^ ] = 1 ,

o por el contrario,

X ^ = 1 2 ( a ^ + a ^ ) , pags ^ = i 2 ( a ^ a ^ ) , [ X ^ , pags ^ ] = i ,

X ^ 2 = norte ^ + 1 2 ( 1 + a ^ 2 + ( a ^ ) 2 ) , pags ^ 2 = norte ^ + 1 2 ( 1 a ^ 2 ( a ^ ) 2 ) .

Considere el espacio Fock | norte := 1 norte ! ( a ^ ) norte | 0 tal que a ^ | 0 = 0 . Considere el estado inicial

| ψ ( 0 ) := 1 2 ( | 0 + | 2 ) , ψ ( 0 ) | = 1 2 ( 0 | + 2 | ) .

Después

| ψ ( t ) = mi i H ^ t | ψ ( 0 ) = 1 2 ( | 0 + mi 2 i t | 2 ) ,

ψ ( t ) | = ψ ( 0 ) | mi i H ^ t = 1 2 ( 0 | + 2 | mi 2 i t ) ,

ψ ( t ) | X ^ | ψ ( t ) = 0 , ψ ( t ) | pags ^ | ψ ( t ) = 0.

Es más,

ψ ( t ) | X ^ 2 | ψ ( t ) = 3 2 + 1 2 porque ( 2 t ) , ψ ( t ) | pags ^ 2 | ψ ( t ) = 3 2 1 2 porque ( 2 t ) ,

porque a ^ 2 | 2 = 2 | 0 . Por lo tanto,

F ( t ) = 9 4 1 2 porque 2 ( 2 t ) ,

que no es constante en el tiempo, y hemos terminado. O alternativamente, podemos completar el contraejemplo sin el uso del argumento de inversión de tiempo anterior simplemente realizando una traducción de tiempo apropiada t t t 0 .

Estaba pensando en tratar de resolver un ejemplo de oscilador armónico yo mismo (porque tengo algunas preguntas más y parece el sistema más simple en el que está sucediendo algo no trivial), pero se me ha adelantado. ¡Gracias!
Aunque hay una cosa que me fastidia. Creo que el cálculo es esencialmente correcto, sin embargo tenemos F ( 0 ) = 1 / 4 lo que significa que minimiza HUP (a menos que esté malinterpretando sus convenciones) y, por lo tanto, ψ ( 0 ) tendría que ser gaussiano, una contradicción con su estado inicial. ¿Hay un pequeño error de cálculo en alguna parte o tengo una falla en mi argumento?
Bien, lo arreglé (espero) :)
Estimado @Marek: Estoy de acuerdo, hubo poderes de 2 falta en tres fórmulas.
+1 Buen ejemplo claro y simple que transmite el punto.
Una cosa que vale la pena señalar: usted dice que la ecuación de Schrödinger no es invariante bajo inversión de tiempo. Es cierto que simplemente sustituyendo t t no es invariante, sino que cambia simultáneamente t t y conjugación compleja ψ ψ deja la ecuación invariante. Eso significa que, para cada solución ψ ( t ) , hay una solución correspondiente ψ ( t ) que "parece" el mismo estado retrocediendo en el tiempo (y en particular tiene los mismos valores esperados para todos los operadores). Eso es lo que la gente quiere decir cuando dice que la ecuación de Schrödinger tiene simetría de inversión de tiempo.
@TedBunn: ¿"Parece" o "es"? En realidad, debería ser "es", según su argumento. ¿Cuál es su posición sobre este asunto?

La ecuación de Schrödinger es simétrica en el tiempo. Por lo tanto, la respuesta es no.

Estoy contigo, pero probablemente sea útil para Marek ver por sí mismo cómo funciona esto en el ejemplo simple para estar convencido de la declaración general.
Sí, este parece un buen argumento para resolver la pregunta original. Pero trae más preguntas :) En particular, la solución de Moshe (crecimiento mínimo tanto hacia el futuro como hacia el pasado) es una especie de rebote. Pero en ambos lados de ese rebote, supongo que la desigualdad se cumpliría. En otras palabras, ¿se mantendría la declaración si permitiéramos estas soluciones rebotantes simples y el tiempo "t = 0"? O para decirlo más claramente: debería haber hecho una pregunta más general sobre cómo se ve la incertidumbre en función del tiempo... Ahora sabemos que no tiene por qué ser monótono, pero tal vez tenga otras buenas propiedades.
No puedo encontrar ni pies ni cabeza en esta oración: en otras palabras, ¿se mantendría la declaración si permitiéramos estas soluciones simples y el tiempo "t = 0". No sé si se puede decir algo interesante en general sobre la evolución temporal de Δ X Δ pags , además de, por supuesto, que está delimitado a continuación.
@Ted: ah, eso no estaba muy claro. La mejor reformulación es probablemente esta: si existe el tiempo t 0 tal que la desigualdad se cumple para todos los tiempos t t 0 . Pero es una pregunta diferente.
Ah, entiendo lo que quieres decir. Esa es una conjetura muy plausible. Apuesto a que es cierto para una partícula libre, pero no lo sé.
Sí, creo que es cierto para una partícula libre. Trabajando en la imagen de Heisenberg, puedes mostrar eso (a menos que haya estropeado los conmutadores, lo cual siempre es posible) d ( X 2 ) / d t 2 = 2 pags 2 / metro 2 . Trabaja en un marco de referencia en el que pags = 0 , de modo que X es constante, y también puede tomarse como cero. Entonces la incertidumbre en X es solo Δ X 2 = X 2 , que tiene una segunda derivada positiva 2 pags 2 / metro 2 . Así que eventualmente comienza a aumentar. En esta situación Δ pags es constante, entonces Δ X Δ pags eventualmente aumenta.
@Ted Bunn; Mi intuición decía lo mismo, pero ¿cómo sabes eso? Δ pags es constante? Tal vez está disminuyendo. Tendría que hacer el cálculo. Pero tiene sentido que si la función de onda se expande, tendrá un gradiente promedio más pequeño y, por lo tanto, tendrá un menor Δ pags .
@Ted: @Carl tiene razón. Como ya he escrito en mi pregunta, el paquete gaussiano gratuito minimiza HUP en todo momento. Pero también sabemos que aumenta la incertidumbre en su posición. Por tanto, la incertidumbre en su impulso debe disminuir. Supongo que esta es una característica general de cada evolución (pero nuevamente no tengo pruebas y bien podría ser que falle por razones triviales).
@Ted, @Carl: está bien, ahora estoy completamente perdido. Por un lado, no veo ningún problema con mi argumento sobre HUP. Por otro lado, entiendo el comentario de Ted sobre Δ pags siendo constante porque el impulso es una integral del movimiento de la partícula libre y, por lo tanto, la distribución del impulso del paquete no puede cambiar en el tiempo. Obviamente, debo estar pasando por alto algo trivial, pero no puedo entender qué es :(
Esta bien, lo tengo. La función evolucionada obviamente ya no es gaussiana. Sólo lo es su densidad de probabilidad pero no su amplitud. Por lo tanto, no hay razón para suponer que todavía minimiza HUP.
Creo que @Marek y yo estamos completamente de acuerdo. Solo para ser explícito, déjame responder la pregunta de @Carl sobre cómo sabemos Δ pags es constante Marek tiene razón: para una partícula libre, pags norte conmuta con el hamiltoniano, por lo que todos los valores esperados pags norte son constantes. Asi que Δ pags 2 = pags 2 pags 2 es constante (De hecho, toda la distribución de probabilidad para pags es constante en el tiempo.) Como resultado, un paquete de ondas gaussianas para una partícula libre no permanece en incertidumbre mínima todo el tiempo. Se propaga en el espacio real mientras permanece igual en el espacio de momento.
@Ted Bunn; En realidad, la referencia que di: demostraciones.wolfram.com/EvolutionOfAGaussianWavePacket muestra que el gaussiano tiene un mínimo / 2 en t=0. En consecuencia, eliminaré el comentario que hice diciendo lo contrario.
Estoy teniendo un momento tan difícil con este concepto. Su declaración me parece incorrecta, y en lugar de discutir, permítame presentar una ecuación para hacer mi argumento. ¿No es cierto que X ( t ) 2 pags ( t ) 2 X ( 0 ) 2 pags ( 0 ) 2 ¿también? El más preciso conocido X y pags están en t = 0 , por lo que con solo esa información, la incertidumbre crece en ambas direcciones de tiempo. Sí, la ecuación es simétrica en el tiempo, pero eso es consistente con que tu respuesta sea incorrecta.
Bueno, esta afirmación no puede ser cierta para todas las soluciones de la ecuación de Schrödinger, ya que no hay nada especial en t = 0 en la ecuacion! Creo que me estoy perdiendo tu punto.
Permítanme ser un poco más específico sobre el punto de mi comentario anterior. Suponga que tiene una solución en la que la incertidumbre crece a medida que se aleja de t = 0 en ambas direcciones. Luego, cambiando su elección (arbitraria) de t = 0 adelante un poco, tendrías una solución en la que la incertidumbre crecía a t = 0 , y cambiando t = 0 un poco hacia atrás, tendría una solución en la que la incertidumbre estaba cayendo en t = 0 .

No. Aquí hay un ejemplo simple donde se encoge:

Tienes una partícula que tiene un 50% de posibilidades de estar a la izquierda yendo a la derecha y un 50% de posibilidades de estar a la derecha yendo a la izquierda. Esto tiene un error macroscópico tanto en la posición como en el momento. Si espera hasta que pase a mitad de camino, tiene un 100% de posibilidades de estar en el medio. Esto tiene un error microscópico en la posición. También habrá solo un cambio microscópico en el impulso. (No estoy del todo seguro de esto, ya que las posibilidades chocan entre sí, pero si solo mira justo antes de eso, o las hace perder un poco, aún funciona).

Como tal, el error en la posición disminuyó significativamente, pero el error en el impulso se mantuvo casi igual.

Marek,

Piense en términos de funciones armónicas y su principio máximo (o teorema del valor medio).

Por simplicidad (y, de hecho, sin pérdida de generalidad), pensemos en términos de una partícula libre, es decir, V ( X , y , z ) = 0 . Cuando el Potencial se desvanece, la ecuación de Schrödinger no es más que una ecuación de Laplace (o ecuación de Poisson, si quiere poner un término fuente). Y, en este caso, puede aplicar el Teorema del Valor Medio (o el Principio del Máximo) y obtener un resultado relacionado con su pregunta: en esta situación, satura la igualdad.

Ahora, si tiene un potencial, puede pensar en términos de un operador de Laplace-Beltrami : todo lo que necesita hacer es 'absorber' el potencial en el término cinético a través de una métrica de Jacobi : gramo ~ = 2 ( mi V ) gramo . (Tenga en cuenta que esto es solo una transformación conforme de la métrica original en su problema). Y, una vez hecho esto, puede girar la misma manivela que hicimos anteriormente, es decir, redujimos el problema al mismo que el anterior. ;-)

Espero que esto ayude un poco.

Lo siento, pero no veo cómo se relaciona esto con la incertidumbre y la evolución del tiempo. ¿Podrías explicar eso?
@Marek: el punto fue explicitado por Qmechanic, en su respuesta anterior. Si aplica lo que dije en la imagen de Schrödinger, obtiene estados en evolución cuya magnitud siempre está limitada por el teorema del valor medio. (Si estuviéramos hablando de operadores acotados, esto podría hacerse riguroso con un poco de análisis funcional).

Una forma física de ver esto es que se conserva el volumen del espacio de fase de un sistema. La mecánica hamiltoniana conserva el volumen de un sistema en su superficie de energía H = E, lo que en mecánica cuántica corresponde a la ecuación de Schrödinger. El volumen del espacio de fase en la superficie de energía del espacio de fase se compone de unidades de volumen 2 norte para las variables de momento y posición más el de la energía i ψ / t   =   H ψ . Esto luego se conserva. Cualquier crecimiento en la incertidumbre Δ pags Δ q   =   / 2 entonces implicaría el crecimiento en el volumen del espacio de fase del sistema. Entonces, esto significaría que hay algún proceso disipativo, o que la dinámica cuántica se reemplaza por alguna ecuación maestra con una pérdida térmica o ambiental de alguna forma. Sin embargo, para una evolución unitaria pura, el volumen del espacio de fase del sistema, o de manera equivalente, el T r ρ y T r ρ 2 son constantes. Esto significa que la relación de incertidumbre es una transformada de Fourier entre observables complementarios que preservan un área   .

-1, esto es completamente irrelevante para mi pregunta. Estoy interesado solo en los estados puros y para esos el volumen de fase es siempre cero y se conserva tan trivialmente. Pero esto no da ninguna información sobre el comportamiento de la incertidumbre.
El volumen que ocupa un sistema en el espacio de fase define la entropía como S   =   k   yo o gramo ( Ω ) por Ω . La entropía de von Neumann
S   =   k   T r   ρ yo o gramo ( ρ ) .
Un estado mixto tiene cada elemento de ρ   =   1 / norte y la huella es ( 1 / norte ) yo o gramo ( 1 / norte )   =   yo o gramo ( norte ) . Un estado puro ocupa una región del espacio de fase que se normaliza a la unidad de volumen, no a cero.

El principio de incertidumbre de Heisenberg gira en torno al efecto Compton, que establece que la longitud de onda (w) es inversamente proporcional a E, p y f. Sin embargo, si uno reúne varias fórmulas de física cuántica, puede crear estas 4 fórmulas y usar una forma de geometría para resolver el Principio de Incertidumbre pero encontrando la ubicación y el momento del electrón (e-) al mismo tiempo mientras puede aproximar dónde está presencia fue, es y será.

[Las fórmulas son de mi creación]

  1. ((h(c/wi)-h(c/wf))/c^2)+m(e-)=m (e- post colisión de fotones)
  2. h(c/wi)-h(c/wf)=E(energía expulsada del fotón cuando colisionó con el electrón) 3.((h(c/wi)-h(c/wf))/c)=p
  3. (h(c/wi)-h(c/wf)+m(e-)c^2=total e- E

Usando una regla 60/30, uno puede encontrar el ángulo al que se lanzará el electrón y dónde estaba antes de que el fotón chocara con él usando la trayectoria original y la corriente para encontrar dónde se cruza. Esto le permitirá encontrar su ubicación.

Si no está de acuerdo con esto, díganos por qué para que pueda mejorarlo.

La fórmula del impulso:ingrese la descripción de la imagen aquí

La energía expulsada del fotón tras la colisión:ingrese la descripción de la imagen aquí